Un disco giratorio con un borde cargado entre la relatividad especial y general

ingrese la descripción de la imagen aquí

Considere que tenemos un disco giratorio (girando, con una velocidad angular constante w, relativa a un observador inercial A). El disco tiene su borde cargado uniformemente con una carga total Q. No hay duda de que A puede detectar bucles de un campo magnético, con su brújula de bolsillo, debido al bucle de corriente eléctrica constante generado por el borde cargado del disco giratorio. A está observando una corriente = Q/T = Q w/ 2 Pi

Mi pregunta es sobre el observador B, en el mismo plano del disco, que gira alrededor del eje del disco con la misma velocidad angular w, relativa a A.

¿B podría detectar un campo magnético con su brújula de bolsillo?

Si observamos el problema desde el punto de vista de la Relatividad Especial, el borde del disco cargado se mueve en relación con B (la misma omega, pero a diferente distancia del eje del disco, por lo que la velocidad relativa), por lo que su brújula puede detectar un campo magnético.

Pero, ¿y si B se considerara a sí misma, según GR, constante en relación con todos los puntos del disco, incluido el borde del disco, considerando que esperaría campos centrífugos y de Coriolis? Debería pensar que su brújula no se mueve en relación con el borde del disco, ¡así que la brújula no debería detectar un campo magnético!

¿Es ella correcta o incorrecta?

¿Qué es ""constante", en relación con el plano del disco"?
@Cryo Observer B y las cargas distribuidas alrededor del borde del plano del disco
Lamento mucho ser exigente, pero es difícil de entender. Supongo que los cargos están distribuidos, pero el observador B no. Las cargas se distribuyen alrededor del borde del disco. Y luego está el 'plan', que probablemente sea plano. Entonces, ¿está diciendo "Observador B y las cargas (distribuidas alrededor del borde del disco)"? ¿Cuál es el papel del plan/avión en esta oración? ¿Está el observador B en el plano que también contiene el disco? ¿Por qué el observador B es 'constante' si no está girando? Si no, ¿en qué se diferencia el observador B del observador A? Dibuja un diagrama, es muy difícil entender lo que quieres decir.
@Cryo Olvídate del avión. Imagina que vivimos en un planeta como la Tierra que gira sobre sí mismo, pero que no tiene un campo magnético intrínseco. Nos enterramos cargados bajo su ecuador. Cada carga está simplemente enterrada bajo la tierra en el ecuador. Si un observador inercial A está ubicado en algún lugar, no en el planeta, verá que el ecuador gira → bucle de corriente eléctrica, causa de las cargas enterradas del ecuador, por lo tanto, A detecta un campo magnético. La pregunta ahora, ¿y nosotros? Si estamos en nuestros hogares en este planeta y las cargas son fijas en relación con nosotros, entonces no hay corriente. ¿Detectaremos algún campo magnético?
No puedo decirlo sin hacer cálculos. Creo que el observador no inercial detectará el campo magnético, pero es una corazonada. Tal vez sería mejor simplificar el problema. Un anillo de carga giratoria en el origen, alineado con el eje z, el observador B en el plano XY gira con la misma velocidad angular que el anillo. Sugeriría encontrar primero el campo eléctrico y magnético para un observador estacionario y luego verificar si B 2 mi 2 / C 2 > 0 (ver en.wikipedia.org/wiki/… ). Si es así, todos los observadores observarán algún campo magnético, ...
si no, se necesita más trabajo
¿Aceptaría una respuesta que involucre una capa esférica giratoria en su lugar? Los campos de marco inercial son mucho más fáciles de derivar para ese, lo que hace posible en principio un cálculo exacto.
@MichaelSeifert, sí, no hay problema. La misma idea del problema supuso que B rotaría, en relación con el observador inercial A, con el mismo vector de velocidad angular w de la esfera cargada.

Respuestas (2)

Empecé a buscar en Google los resultados básicos para un caparazón esférico giratorio y descubrí que alguien se me había adelantado en este mismo cálculo:

Campos electromagnéticos de una capa de carga giratoria , Kirk T. McDonald

En el artículo, el autor calcula los campos en un marco inercial y luego proporciona una transformación a un marco giratorio para encontrar los campos magnéticos y eléctricos observados en ese marco. El autor trata el caso más general donde un caparazón con carga q y radio a gira con velocidad angular ω ω , mientras que el marco giratorio gira a ω ω ; el caso solicitado por el OP es simplemente el caso ω ω = ω ω .

El resultado dentro del caparazón es en realidad notablemente simple: en el límite de rotaciones lentas, el campo magnético medido por un observador corrotatorio dentro del caparazón es exactamente el mismo que el campo magnético medido por un observador inercial dentro del caparazón. La razón de esto es bastante sencilla: la transformación entre los campos para dos marcos inerciales, en el límite de velocidades lentas, es

B B v C mi .
Dentro de la concha, mi = 0 y entonces B = B . Por lo tanto, si un observador inercial mide un campo magnético dentro de la capa, también lo hará el observador corrotatorio.

La razón por la que todavía hay un campo dentro del caparazón, aunque el observador giratorio no vea corrientes, es simplemente que las ecuaciones de Maxwell no se mantienen en un marco de referencia giratorio. El documento vinculado anteriormente presenta las ecuaciones de Maxwell en un marco de referencia que gira (lentamente) en las Ecs. (31–34) de las notas vinculadas anteriormente. En ausencia de fuentes ligadas, y en el caso estacionario (como el que tenemos aquí), se convierten en

mi = 4 π [ ρ v C 2 j + ω ω B 2 π C ] , × B = 4 π C [ j + v ρ + ω ω × mi 4 π ω 4 π mi ϕ ] ,
dónde v = ω ω × r y ϕ (Las otras dos ecuaciones de Maxwell siguen siendo válidas; las notas tienen ω ω en lugar de ω ω , pero creo que esto es un error.) En el marco giratorio, mientras j = 0 , todavía hay contribuciones al campo magnético de los otros términos en el lado derecho de la última ecuación y, por lo tanto, el campo magnético no desaparece. (Creo que los dos últimos términos se anulan en el presente caso, ya que corresponden a la tasa de cambio de mi con respecto a t ; pero el ρ v el término ciertamente no.)

Las referencias en el artículo enlazado también pueden ser de interés; en particular, es posible que desee consultar las notas del mismo autor sobre electrodinámica en marcos giratorios , así como los siguientes artículos:

  • LI Schiff, "Una cuestión de relatividad general", Proc. Nat. Academia ciencia 25, 391 (1939)
  • CT Ridgely, "Aplicación de la electrodinámica relativista a un medio material giratorio", Am. J. física. 66, 114 (1998)
  • CT Ridgely, "Aplicación de tensores electromagnéticos covariantes versus contravariantes a medios giratorios", Am. J. física. 67, 414 (1999)
Mi intuición dice que este campo magnético residual se debe a que las ecuaciones de Maxwell no funcionan en marcos de referencia no inerciales. Creo firmemente que el observador en movimiento conjunto no debería notar ningún campo magnético en el caso general, ya que no hay una "corriente relativa" desde el punto de vista de GR. Además, no entendí la declaración del autor "se observa un campo magnético distinto de cero en ese marco, que se puede atribuir a las corrientes "ficticias". Se observa un campo magnético distinto de cero en ese marco, que se puede atribuir a la " “corrientes” ficticias. ¡¿Qué es la corriente “ficticia”?!
@AhmedKamalKassem: Creo que se refiere a los términos adicionales que aparecen en el lado derecho de la Ley de Gauss y la Ley de Ampere, que escribí anteriormente (para un caso especial). La analogía es con "fuerzas ficticias" que aparecen cuando Las Leyes de Newton están escritas en un marco no inercial. El autor de las notas que vinculé parece haber obtenido la terminología del artículo de Schiff al que me refiero al final de la respuesta.

Entonces, calcularé la respuesta para un anillo giratorio de carga de radio. R , que se puede describir por la densidad de corriente:

j = I 0 2 π R . d ϕ ( pecado ϕ porque ϕ 0 ) d ( 3 ) ( r R ( porque ϕ pecado ϕ 0 ) )

Dónde I es la corriente en el anillo. Esto es para un observador inercial y está en coordenadas cartesianas. Esto se puede introducir en la ley de Biot-Savart para encontrar el campo magnético ( m 0 es la permeabilidad al vacío):

B ( r ) = m 0 4 π d 3 r j ( r ) × ( r r ) | r r | 3

Para un observador (A) sentado en el plano XY, a una distancia ρ lejos del origen, el campo magnético estará polarizado en z:

B z ( ρ ) = I m 0 4 π 0 2 π R d ϕ . R ρ porque ϕ ( R 2 + ρ 2 2 R ρ porque ϕ ) 3 / 2 = I m 0 4 π R k ( ρ , R )

Dónde

k ( ρ , R ) = 0 2 π d ϕ . R 2 ( R ρ porque ϕ ) ( R 2 + ρ 2 2 R ρ porque ϕ ) 3 / 2

es simplemente una forma abreviada de la integral sin unidades que no sé cómo evaluar (aparte de en el límite o numéricamente).

Sea la carga completa del bucle q , sea la velocidad angular de rotación ω . Entonces I = q ω / 2 π y:

B z ( ρ ) = q ω m 0 8 π 2 R k ( ρ , R )

A continuación, el campo eléctrico del anillo de densidad de carga:

ρ = q 2 π 0 2 π d ϕ d ( 3 ) ( r R ( porque ϕ pecado ϕ 0 ) )

para observador A el campo eléctrico será puramente radial ( ϵ 0 es la permitividad del vacío):

mi ρ ( ρ ) = q / 2 π 4 π ϵ 0 0 2 π d ϕ . ρ R porque ϕ ( R 2 + ρ 2 2 R ρ porque ϕ ) 3 / 2

de nuevo, introduciendo:

S ( ρ , R ) = 0 2 π d ϕ . R 2 ( ρ R porque ϕ ) ( R 2 + ρ 2 2 R ρ porque ϕ ) 3 / 2

el campo eléctrico se convierte

mi ρ ( ρ ) = q 8 π 2 R 2 ϵ 0 S ( ρ , R )

Ahora considera:

B z mi ρ / C = ω R C × k ( ρ , R ) S ( ρ , R )

Dónde C es la velocidad de la luz. Jugando con r R muestra que para k / S 1 , por lo que la cantidad clave aquí es ω R / C que es la relación entre la velocidad con la que se mueven las cargas en la espira y la velocidad de la luz.

Claramente ω R / C < 1 , pero para grandes velocidades angulares, en principio, es posible tener C B z / mi ρ > 1 . Entonces, todos los observadores que pasen por ese punto en el espacio observarán un campo magnético distinto de cero. La lógica se basa en B 2 mi 2 / C 2 invariante del campo electromagnético

En condiciones normales, los campos eléctricos y magnéticos vistos por el observador inercial serán, asumiendo que el observador está desplazado desde el origen a lo largo del eje x:

mi X = mi B z ω R C mi

A partir de aquí, debería poder aplicar transformadas de Lorentz para aproximar el campo electromagnético en el marco de inercia instantáneo del observador giratorio B


Noté esa proporción k / S diverge para r R , por lo que si el observador está sentado en el bucle, probablemente podría tener una situación C B z / mi ρ > 1 incluso para velocidades de rotación relativamente suaves